1 Introduction

In this paper, we deal with the following kind of problem:

$$\begin{aligned} \left\{ \begin{aligned}&-\mathrm{div} (M(x) \nabla u )+ \gamma u^p= B \frac{|\nabla u|^q}{u^\theta }+f\ \ \mathrm{in}\ \Omega ,\\&u> 0\ \ \mathrm{in}\ \Omega ,\\&u=0\ \ \mathrm{on}\ {\partial \Omega },\\ \end{aligned} \right. \end{aligned}$$
(1.1)

where \(\Omega \) is any bounded open subset of \(\mathbb {R}^N\), \(N>2\), \(\displaystyle {M: \Omega \times \mathbb {R}\rightarrow \mathbb {R}^{N^2}}\) is a bounded and measurable matrix such that there exist \(\alpha , \beta > 0\) satisfying:

$$\begin{aligned} \alpha |\xi |^2 \le M(x)\xi \xi , \quad |M(x)|\le \beta \quad \text {a.e.} \ x \in \Omega , \ \forall \xi \in \mathbb {R}^N. \end{aligned}$$
(1.2)

We assume that:

$$\begin{aligned} 0<\theta \le 1, \ \gamma> 0, \ B> 0, \ 1\le q<2, \ \text {and} \ p> \frac{q}{2-q}. \end{aligned}$$
(1.3)

We suppose moreover that f is a nonnegative function, such that \(f\not \equiv 0\) and that:

$$\begin{aligned} f \in L^1(\Omega ). \end{aligned}$$
(1.4)

The main features in dealing with this class of problems are the facts that the lower order term has a singularity in u and the data f belong to \(L^1(\Omega )\). For \(B<0\), \(\gamma =0\) and the lower order term satisfies a quadratic growth assumption with respect to the gradient and depends continuously on u, the problems like (1.1) have been exhaustively studied in the literature, see for instance [4, 7, 10], while the case of singular lower order term at \(u=0\) having a quadratic growth with respect to the gradient was considered in [3, 5, 14]. We refer also to [2] where the problem (1.1) is deeply studied in the case \(\gamma =0\), \(q=2\), \(\theta =1\), and f belongs to \(L^m(\Omega )\), \(m>1\).

The existence and regularity results of problems (1.1) have been proved in [12] in the case where the lower order term is nonsingular and the growth of the gradient is superlinear and f belongs to \(L^m(\Omega )\), \(m \ge 1\) (see also [15, 16]). In addition, the problem of existence and regularity of solutions have been investigated in [11] when \(\gamma =0\), \(q=1\) and f belongs to \(L^m(\Omega )\), \(m>1\). We also quote the paper [1] in which the existence and nonexistence results related to (1.1) have been obtained using a comparison and a priori estimates if \(\gamma =0\), \(1<q \le 2\), and f belongs to \(L^m(\Omega )\), \(m \ge 1\).

Our aim is to prove the existence of solution of problem (1.1) under our assumptions (1.2)–(1.4). Note that the presence of the lower order term \(\displaystyle {\gamma u^p}\) is crucial in the sense that it guarantees the existence of solution when the data f belong only in \(L^1(\Omega )\). Our main result is the following

Theorem 1.1

Assume that (1.2)–(1.4) hold true. Then, there exists a solution u for (1.1), with \(u>0\) in \(\Omega \), in the sense that:

\(u \in W^{1, r}_0(\Omega ) \cap L^p(\Omega )\) for every \(1\le r<\max {\{\frac{N}{N-1},\frac{2p}{p+1} \}}\), \(\frac{|\nabla u|^q}{u^\theta } \in L_\mathrm{loc}^1(\Omega )\) and that:

$$\begin{aligned} \int _{\Omega }M(x)\nabla u \nabla \varphi + \gamma \int _{\Omega } u^p \varphi = \int _{\Omega } B \frac{|\nabla u|^q}{u^\theta }\varphi +\int _{\Omega }f \varphi , \quad \forall \varphi \in C^1_c(\Omega ).\quad \end{aligned}$$
(1.5)

Moreover, if \(0<\theta \le \frac{q}{q+1}\), then \(\frac{|\nabla u|^q}{u^\theta }\) belongs to \(L^1(\Omega )\).

Remark 1.2

If \(0<\theta \le \frac{q}{q+1}\), then we can use, as test functions in the formulation (1.5), not only functions in \(C^1_c(\Omega )\), but also functions in \(W^{1, s}_0(\Omega )\), \(s>\max {\{N,\frac{2p}{p-1} \}}\).

2 Approximation of Problem (1.1)

We will prove the existence of solutions of problem (1.1) by a standard approximation procedure which avoids singularities. To this end, we consider for \( n \in \mathbb {N}\) the following approximate problem:

$$\begin{aligned} \left\{ \begin{aligned}&-\mathrm{div} (M(x)\nabla u_n )+\gamma |u_n|^{p-1}u_n=B\frac{|\nabla u_n|^q}{(1+\frac{1}{n}|\nabla u_n|^q)(|u_n|+\frac{1}{n})^\theta }+f_n\ \ \mathrm{in}\ \Omega ,\\&u_n\ge 0\ \ \mathrm{in}\ \Omega ,\\&u_n=0\ \ \mathrm{on}\ {\partial \Omega },\\ \end{aligned} \right. \nonumber \\ \end{aligned}$$
(2.1)

where \(f_n =\frac{f}{1+\frac{1}{n}f}\). Since the right-hand side of (2.1) is bounded, the existence of a bounded (see [18]) weak solutions \(u_n \in H_0^{1}(\Omega )\) is a consequence of the classical results in [17] (see also [13]). Moreover, \(u_n \ge 0\), since the right-hand side of (2.1) is positive. Now, let z be the unique weak solution of:

$$\begin{aligned} \left\{ \begin{aligned}&-\mathrm{div} (M(x)\nabla z )+\gamma z^p=\frac{f}{1+f}\ \ \mathrm{in}\ \Omega ,\\&z=0\ \ \mathrm{on}\ {\partial \Omega }.\\ \end{aligned} \right. \end{aligned}$$
(2.2)

Since \(\frac{f}{1+f} \ge 0\) and \(\int _{0} (s^{p+1})^{\frac{-1}{2}}= \infty \) with \(p \ge 1\), the strong maximum principle (see [19]) implies that for every \(\omega \subset \subset \Omega \), there exists \(c_\omega \), such that the solution z of (2.2) satisfies \(z \ge c_{\omega }> 0 \ \text {in} \ \omega \). Since \(\frac{f}{1+\frac{1}{n}f}\ge \frac{f}{1+f}\), one has:

$$\begin{aligned} -\mathrm{div} (M(x)\nabla u_n )+\gamma u_n^p \ge -\mathrm{div} (M(x)\nabla z )+\gamma z^p; \end{aligned}$$
(2.3)

so, using \(-(u_n-z)^{-}\) as test function in (2.3) with \((u_n-z)^{-}= -(u_n-z)\chi _{\{u_n<z\}}\), we obtain:

$$\begin{aligned} \int _{\{u_n<z\}} |\nabla (u_n-z)|^2- \gamma \int _{\Omega } (u_n^p-z^p) (u_n-z)^{-} \le 0. \end{aligned}$$
(2.4)

Dropping the second nonnegative term in the left-hand side of (2.4), it follows that \(u_n \ge z\). Therefore:

$$\begin{aligned} u_n \ge c_{\omega }> 0 \ \text {in} \ \omega , \quad \forall \omega \subset \subset \Omega . \end{aligned}$$
(2.5)

Hereafter, we will make use of two truncation functions \(T_k\) and \(G_k\): for every \(k \ge 0\) and \(r \in \mathbb {R}\), let:

$$\begin{aligned} T_k(r)=\left\{ \begin{array}{ll} r &{} \mathrm{if} \ \ |r|\le k,\\ k\frac{r}{|r|} &{} \mathrm{if} \ \ |r|>k,\\ \end{array} \right. \quad G_k(r)=r-T_k(r). \end{aligned}$$

For the sake of simplicity, we will use when referring to the integrals the following notation:

$$\begin{aligned} \int _\Omega f = \int _\Omega f(x) \, \mathrm{d}x. \end{aligned}$$

Finally, throughout this paper, C will indicate any positive constant which depends only on data and whose value may change from line to line and sometimes in the same line.

3 Existence Result

In the next lemma, we state some a priori estimates on the solution \(u_n\) and on the lower order term of the approximate problem (2.1).

Lemma 3.1

Assume (1.2)–(1.4). Then, the sequence \(u_n\) is bounded in \( W^{1, r}_0(\Omega ) \cap L^p(\Omega )\) for every \(1\le r<\max {\{\frac{N}{N-1},\frac{2p}{p+1} \}}\) and \(\frac{|\nabla u_n|^q}{u_n^\theta }\) is bounded in \(L_\mathrm{loc}^1(\Omega )\).

Proof

Let \(\lambda >1\), and choose \(1-(1+G_1(u_n))^{1-\lambda }\) as test function in (2.1), using (1.2) and the fact that \(f_n \le f\), we thus have:

$$\begin{aligned}&(\lambda -1) \alpha \int _{\{ u_n\ge 1\}}\frac{|\nabla u_n|^2}{u_n^\lambda }+ \gamma \int _{\{ u_n\ge 1\}} u_n^p(1-u_n^{1-\lambda })\nonumber \\&\quad \le B\int _{\{ u_n\ge 1\}}\frac{|\nabla u_n|^q}{u_n^\theta }+ \int _{\Omega }f, \end{aligned}$$
(3.1)

using Young inequality, we obtain:

$$\begin{aligned}&B\int _{\{ u_n\ge 1\}}\frac{|\nabla u_n|^q}{u_n^\theta }\le B\int _{\{ u_n\ge 1\}}|\nabla u_n|^q= B\int _{\{ u_n\ge 1\}}\frac{|\nabla u_n|^q}{u_n^{\frac{q\lambda }{2}}} u_n^{\frac{q\lambda }{2}}\\&\quad \le \frac{(\lambda -1) \alpha }{2}\int _{\{ u_n\ge 1\}}\frac{|\nabla u_n|^2}{u_n^\lambda }+C\int _{\{ u_n\ge 1\}} u_n^{\frac{q\lambda }{2-q}}, \end{aligned}$$

which implies from (3.1) that:

$$\begin{aligned}&\frac{(\lambda -1) \alpha }{2}\int _{\{ u_n\ge 1\}}\frac{|\nabla u_n|^2}{u_n^\lambda }+\gamma \int _{\{ u_n\ge 1\}} u_n^p(1-u_n^{1-\lambda })\nonumber \\&\quad \le C\int _{\{ u_n\ge 1\}} u_n^{\frac{q\lambda }{2-q}}+\int _{\Omega }f. \end{aligned}$$
(3.2)

Next, we choose \(T> 1\), such that \(1-T^{1-\lambda }= \frac{1}{2}\), we have:

$$\begin{aligned} \frac{1}{2}\int _{\{ u_n> T\}} u_n^p \le \int _{\{ u_n> T\}} u_n^p(1-u_n^{1-\lambda }) \le \int _{\{ u_n\ge 1\}} u_n^p(1-u_n^{1-\lambda }), \end{aligned}$$

that is:

$$\begin{aligned} \frac{1}{2}\int _{\{ u_n\ge 1\}} u_n^p= & {} \frac{1}{2}\int _{\{ 1 \le u_n\le T\}} u_n^p+\frac{1}{2}\int _{\{ u_n> T\}} u_n^p\\\le & {} \frac{1}{2} T^p \mathrm{meas}(\Omega )+\int _{\{ u_n\ge 1\}} u_n^p(1-u_n^{1-\lambda }). \end{aligned}$$

Hence, from (3.2), we deduce that:

$$\begin{aligned}&\frac{(\lambda -1) \alpha }{2}\int _{\{ u_n\ge 1\}}\frac{|\nabla u_n|^2}{u_n^\lambda }+\frac{\gamma }{2} \int _{\{ u_n\ge 1\}} u_n^p\\&\quad \le C\int _{\{ u_n\ge 1\}} u_n^{\frac{q\lambda }{2-q}}+C. \end{aligned}$$

Choosing \(\lambda \) such that \(\frac{q\lambda }{2-q}<p\), that is \(1<\lambda < \frac{p(2-q)}{q}\). Note that this choice of \(\lambda \) is possible, since \(\frac{p(2-q)}{q}>1\). Therefore:

$$\begin{aligned} \int _{\{ u_n\ge 1\}}\frac{|\nabla u_n|^2}{u_n^\lambda } \le C, \end{aligned}$$
(3.3)

and

$$\begin{aligned} \int _{\Omega } u_n^p \le C. \end{aligned}$$
(3.4)

Now, we choose \(\varepsilon < \frac{1}{n}\) and use \((T_1(u_n)+\varepsilon )^\theta -\varepsilon ^\theta \) as test function, dropping the positive term and using (1.2), we have:

$$\begin{aligned}&\alpha \theta \int _{\Omega }\frac{|\nabla T_1(u_n)|^2}{(T_1(u_n)+\varepsilon )^{1-\theta }} \le B\int _{\Omega }\frac{|\nabla u_n|^q}{(u_n+\frac{1}{n})^\theta }(T_1(u_n)+\varepsilon )^\theta \nonumber \\&\qquad +\int _{\Omega } f_n (T_1(u_n)+\varepsilon )^\theta \nonumber \\&\quad \le B\int _{\{ u_n\ge 1\}} |\nabla u_n|^q+ B\int _{\{ u_n<1\}}|\nabla u_n|^q+ (1+\varepsilon )^\theta \int _{\Omega } f. \end{aligned}$$
(3.5)

Using Young inequality together with (3.3) and (3.4) and the fact that \(\frac{q\lambda }{2-q}<p\) yield that:

$$\begin{aligned} \int _{\{ u_n\ge 1\}} |\nabla u_n|^q= & {} \int _{\{ u_n\ge 1\}}\frac{|\nabla u_n|^q}{u_n^{\frac{\lambda q}{2}}} u_n^{\frac{\lambda q}{2}}\\\le & {} C\int _{\{ u_n\ge 1\}}\frac{|\nabla u_n|^2}{u_n^\lambda }+ C\int _{\Omega } u_n^p \le C. \end{aligned}$$

Then, from (3.5) and the above estimate, using again young inequality, we obtain:

$$\begin{aligned}&\alpha \theta \int _{\Omega }\frac{|\nabla T_1(u_n)|^2}{(T_1(u_n)+\varepsilon )^{1-\theta }} \le B \int _{\Omega }\frac{|\nabla T_1(u_n)|^q}{(T_1(u_n)+\varepsilon )^{\frac{q}{2}(1-\theta )}}(T_1(u_n)+\varepsilon )^{\frac{q}{2}(1-\theta )}\\&\qquad +(1+\varepsilon )^\theta \int _{\Omega } f+C \\&\quad \le \frac{\alpha \theta }{2} \int _{\Omega }\frac{|\nabla T_1(u_n)|^2}{(T_1(u_n)+\varepsilon )^{1-\theta }}+C(1+\varepsilon )^{\frac{q}{2-q}(1-\theta )}+(1+\varepsilon )^\theta \int _{\Omega } f+C, \end{aligned}$$

and so

$$\begin{aligned} \int _{\Omega }\frac{|\nabla T_1(u_n)|^2}{(T_1(u_n)+\varepsilon )^{1-\theta }}\le C\Bigg ((1+\varepsilon )^{\frac{q}{2-q}(1-\theta )}+(1+\varepsilon )^\theta \Bigg ), \end{aligned}$$

which gives:

$$\begin{aligned} \int _{\Omega }|\nabla T_1(u_n)|^2= & {} \int _{\Omega }\frac{|\nabla T_1(u_n)|^2}{(T_1(u_n)+\varepsilon )^{1-\theta }}(T_1(u_n)+\varepsilon )^{1-\theta }\\\le & {} C (1+\varepsilon )^{1-\theta }\Bigg ((1+\varepsilon )^{\frac{q}{2-q}(1-\theta )}+(1+\varepsilon )^\theta \Bigg ). \end{aligned}$$

Letting \(\varepsilon \) tends to 0 yields that:

$$\begin{aligned} \int _{\Omega }|\nabla T_1(u_n)|^2 \le C. \end{aligned}$$
(3.6)

Combining (3.3) and (3.6), we have:

$$\begin{aligned} \int _{\Omega }\frac{|\nabla u_n|^2}{(1+u_n)^\lambda } \le \int _{\{ u_n\ge 1\}}\frac{|\nabla u_n|^2}{u_n^\lambda }+\int _{\Omega }|\nabla T_1(u_n)|^2 \le C, \end{aligned}$$
(3.7)

which holds for every \(\lambda >1\). Now, we proceed as in [12] (Proposition A.1 and A.2 in the Appendix A). Let \(1 \le r<2\), using the estimate (3.7) together with Hölder inequality, we obtain:

$$\begin{aligned} \int _{\Omega }|\nabla u_n|^r \le \int _{\Omega }\frac{|\nabla u_n|^r}{(1+u_n)^{\frac{r\lambda }{2}}}(1+u_n)^{\frac{r\lambda }{2}}\le C \Bigg (\int _{\Omega }(1+u_n)^{\frac{r\lambda }{2-r}}\Bigg )^{1-\frac{r}{2}}. \end{aligned}$$
(3.8)

Sobolev inequality implies that:

$$\begin{aligned} \Bigg (\int _{\Omega } u_n^{r^*}\Bigg )^{\frac{r}{r^*}}\le C\Bigg (\int _{\Omega }(1+u_n)^{\frac{r\lambda }{2-r}}\Bigg )^{1-\frac{r}{2}}. \end{aligned}$$

Choosing r such that \(r^*=\frac{rN}{N-r}=\frac{r\lambda }{2-r}\) gives that \(r=\frac{N(2-\lambda )}{N-\lambda }\). Notice that \(\frac{r}{r^*} >1-\frac{r}{2}\) (which is equivalent to \(N>2\)), and since \(\lambda >1\), it follows that:

$$\begin{aligned} \int _{\Omega }|\nabla u_n|^r\le C, \quad \forall \ 1 \le r< \frac{N}{N-1}. \end{aligned}$$
(3.9)

On the other hand, starting from (3.8) and thanks to (3.4), noticing that \(\frac{r\lambda }{2-r}\le p\) is equivalent to \(r\le \frac{2p}{p+\lambda }\), and since \(\lambda >1\), we thus obtain:

$$\begin{aligned} \int _{\Omega }|\nabla u_n|^r\le C, \quad \forall \ 1 \le r< \frac{2p}{p+1}. \end{aligned}$$
(3.10)

Combining (3.9) and (3.10) to conclude that:

$$\begin{aligned} \int _{\Omega }|\nabla u_n|^r\le C, \quad \forall \ 1 \le r< \max \Bigg \{\frac{N}{N-1}, \ \frac{2p}{p+1} \Bigg \}. \end{aligned}$$
(3.11)

Recalling (2.5), estimate (3.11) and by means of Hölder inequality, it follows for every \(\omega \subset \subset \Omega \) that:

$$\begin{aligned} \int _{\omega }\frac{|\nabla u_n|^q}{u_n^\theta } \le \frac{\mathrm{meas}(\Omega )^{\frac{r-q}{r}}}{c_\omega ^\theta }\Vert u_n \Vert ^q_{W_0^{1, r}(\Omega )} \le C. \end{aligned}$$
(3.12)

\(\square \)

Now, we prove the global boundedness of \(\frac{|\nabla u_n|^q}{u_n^\theta }\) in \(L^1(\Omega )\) in the case where \(0<\theta \le \frac{q}{q+1}\).

Lemma 3.2

Assume (1.2)–(1.4). If \(0<\theta \le \frac{q}{q+1}\), then \(\frac{|\nabla u_n|^q}{u_n^\theta }\) is bounded in \(L^1(\Omega )\).

Proof

To prove Lemma 3.2 we have to distinguish between three cases.

First case. Assume that \(0<\theta <\frac{1}{2}\). We choose \(\varepsilon < \frac{1}{n}\) and use \((T_1(u_n)+\varepsilon )^{1-2\theta }-\varepsilon ^{1-2\theta }\) as test function in (2.1), using (1.2), dropping the positive term, we have:

$$\begin{aligned}&\alpha (1-2\theta ) \int _{\Omega }\frac{|\nabla T_1(u_n)|^2}{(T_1(u_n)+\varepsilon )^{2\theta }} \le B(1+\varepsilon )^{1-2\theta }\\&\quad \times \int _{\Omega }\frac{|\nabla T_1(u_n)|^q}{(T_1(u_n)+\varepsilon )^{q\theta }}(T_1(u_n)+\varepsilon )^{(q-1)\theta }\\&\quad +B(1+\varepsilon )^{1-2\theta }\int _{\{ u_n\ge 1\}} |\nabla u_n|^q+(1+\varepsilon )^{1-2\theta }\int _{\Omega } f, \end{aligned}$$

using (3.11) and Young inequality, we obtain:

$$\begin{aligned} \frac{\alpha (1-2\theta )}{2} \int _{\Omega }\frac{|\nabla T_1(u_n)|^2}{(T_1(u_n)+\varepsilon )^{2\theta }} \le C(\varepsilon ), \end{aligned}$$

where \(C(\varepsilon )\) is such that \({\lim _{\varepsilon \rightarrow 0}} \ C(\varepsilon ) <+\infty .\) That is:

$$\begin{aligned} \int _{\Omega }\frac{|\nabla T_1(u_n)|^2}{(T_1(u_n)+\varepsilon )^{2\theta }} \le C. \end{aligned}$$
(3.13)

Using (3.11), (3.13), and Hölder inequality, we obtain:

$$\begin{aligned} \int _{\Omega }\frac{|\nabla u_n|^q}{(u_n+\varepsilon )^{\theta }}\le & {} \int _{\Omega }\frac{|\nabla T_1(u_n)|^q}{(T_1(u_n)+\varepsilon )^{\theta }}+\int _{\{ u_n\ge 1\}} |\nabla u_n|^q \\\le & {} C+\int _{\Omega }\frac{|\nabla T_1(u_n)|^q}{(T_1(u_n)+\varepsilon )^{q\theta }}(T_1(u_n)+\varepsilon )^{(q-1)\theta } \\\le & {} C+(1+\varepsilon )^{(q-1)\theta }\int _{\Omega }\frac{|\nabla T_1(u_n)|^q}{(T_1(u_n)+\varepsilon )^{q\theta }} \\\le & {} C+\Bigg ( \int _{\Omega }\frac{|\nabla T_1(u_n)|^2}{(u_n+\varepsilon )^{2\theta }} \Bigg )^{\frac{q}{2}} (1+\varepsilon )^{(q-1)\theta } meas(\Omega )^{\frac{2-q}{q}} \le C. \end{aligned}$$

Thanks to Fatou’s Lemma, letting \(\varepsilon \) tends to zero, we deduce that:

$$\begin{aligned} \int _{\Omega }\frac{|\nabla u_n|^q}{u_n^{\theta }} \le C. \end{aligned}$$

Second case. Assume that \(\theta =\frac{1}{2}\), we choose \(\varepsilon < \frac{1}{n}\) and use \(\log (T_1(u_n)+\varepsilon )-\log (\varepsilon )\) as test function, using (1.2), dropping the positive term, we obtain:

$$\begin{aligned} \alpha \int _{\Omega }\frac{|\nabla T_1(u_n)|^2}{T_1(u_n)+\varepsilon }\le & {} B (1+\varepsilon )^{\frac{1}{2}}\int _{\Omega }|\nabla T_1(u_n)|^q \\&+ B \log (1+\varepsilon )\int _{\Omega } |\nabla G_1(u_n)|^q+\log (1+\varepsilon )\int _{\Omega } f, \end{aligned}$$

Using (3.3) and (3.6), we thus have:

$$\begin{aligned} \int _{\Omega }\frac{|\nabla T_1(u_n)|^2}{T_1(u_n)+\varepsilon } \le C ((1+\varepsilon )^{\frac{1}{2}}+\log (1+\varepsilon )), \end{aligned}$$

and by Hölder inequality, it follows that:

$$\begin{aligned} \int _{\Omega }\frac{|\nabla T_1(u_n)|^q}{(T_1(u_n)+\varepsilon )^{\frac{1}{2}}}= & {} \int _{\Omega }\frac{|\nabla T_1(u_n)|^q}{(T_1(u_n)+\varepsilon )^{\frac{q}{2}}} (T_1(u_n)+\varepsilon )^{\frac{q-1}{2}}\\\le & {} (1+\varepsilon )^{\frac{q-1}{2}} \mathrm{meas}(\Omega )^{\frac{2-q}{q}} \Bigg (\int _{\Omega }\frac{|\nabla T_1(u_n)|^2}{T_1(u_n)+\varepsilon } \Bigg )^{\frac{q}{2}} \\\le & {} C(1+\varepsilon )^{\frac{q-1}{2}} \mathrm{meas}(\Omega )^{\frac{2-q}{q}} \Bigg ( (1+\varepsilon )^{\frac{1}{2}}+\log (1+\varepsilon )\Bigg )^{\frac{q}{2}}. \end{aligned}$$

Thanks to Fatou’s Lemma, letting \(\varepsilon \) tends to zero, we deduce that:

$$\begin{aligned} \int _{\Omega }\frac{|\nabla u_n|^q}{u_n^{\theta }} \le C. \end{aligned}$$

Third case. Assume that \(\frac{1}{2}<\theta <\frac{q}{q+1}\), we choose \(\varepsilon < \frac{1}{n}\), using \((T_1(u_n)+\varepsilon )^{2\theta -1}-\varepsilon ^{2\theta -1}\) and reasoning as above, we obtain:

$$\begin{aligned} \int _{\Omega }\frac{|\nabla T_1(u_n)|^2}{(T_1(u_n)+\varepsilon )^{2(1-\theta )}} \le C; \end{aligned}$$
(3.14)

using Hölder inequality together with (3.14) yields:

$$\begin{aligned} \int _{\Omega }\frac{|\nabla T_1(u_n)|^q}{(T_1(u_n)+\varepsilon )^{\theta }}= & {} \int _{\Omega }\frac{|\nabla T_1(u_n)|^q}{(T_1(u_n)+\varepsilon )^{q(1-\theta )}} (T_1(u_n)+\varepsilon )^{q(1-\theta )-\theta } \\\le & {} (1+\varepsilon )^{q-(q+1)\theta } \mathrm{meas}(\Omega )^{\frac{2-q}{q}} \Bigg (\int _{\Omega }\frac{|\nabla T_1(u_n)|^2}{(T_1(u_n)+\varepsilon )^{2(1-\theta )}}\Bigg )^{\frac{q}{2}}\\\le & {} C (1+\varepsilon )^{q-(q+1)\theta }. \end{aligned}$$

By Fatou’s Lemma, letting \(\varepsilon \) tends to zero, we deduce that:

$$\begin{aligned} \int _{\Omega }\frac{|\nabla u_n|^q}{u_n^{\theta }} \le C. \end{aligned}$$

\(\square \)

Remark 3.3

Thanks to estimates (3.4) and (3.11), it follows that there exists a function \(u \in W^{1, r}_0(\Omega ) \cap L^p(\Omega )\), such that up to a subsequence, \(u_n\) converges to u weakly in \(W^{1, r}_0(\Omega )\) and \(L^p(\Omega )\) and a.e. in \(\Omega \).

We prove now the following convergence result.

Proposition 3.4

Under assumptions (1.2)–(1.4), we have:

$$\begin{aligned} u_n^p \rightarrow u^p \ \ \text {strongly in} \ \ L^1(\Omega ). \end{aligned}$$

Proof

We take \(T_1(u_n-T_h(u_n))\) as test function in (2.1), dropping the positive term, using (1.2), and we then have:

$$\begin{aligned}&\alpha \int _{\{ h \le u_n\le h+1\}}|\nabla u_n|^2+\gamma \int _{\{ u_n \ge h+1\}} u_n^p\\&\quad \le B\int _{\{ h \le u_n\le h+1\}}|\nabla u_n|^q+B\int _{\{ u_n > h+1\}}|\nabla u_n|^q+\int _{\{ u_n \ge h\}} f, \end{aligned}$$

which implies using (3.11), Young together with Hölder inequalities that:

$$\begin{aligned}&\frac{\alpha }{2} \int _{\{ h \le u_n\le h+1\}}|\nabla u_n|^2+\gamma \int _{\{ u_n \ge h+1\}} u_n^p \\&\quad \le C \mathrm{meas}{\{ u_n> h\}}^{1-\frac{q}{2}}+B\Vert u_n \Vert ^q_{W_0^{1, r}(\Omega )} \mathrm{meas}{\{ u_n> h\}}^{\frac{r-q}{r}}+\int _{\{ u_n \ge h\}} f, \\&\quad \le C \mathrm{meas}{\{ u_n> h\}}^{1-\frac{q}{2}}+C \mathrm{meas}{\{ u_n > h\}}^{\frac{r-q}{r}}+\int _{\{ u_n \ge h\}} f. \end{aligned}$$

Letting \(n \rightarrow +\infty \) and then \(h \rightarrow +\infty \), we obtain:

$$\begin{aligned} \int _{\{ u_n \ge h+1\}} u_n^p \le w(n, h), \end{aligned}$$
(3.15)

where w(nh) tends to zero when \(n \rightarrow +\infty \) and \(h \rightarrow +\infty \).

Let E be a measurable subset of \(\Omega \), and we have:

$$\begin{aligned} \int _{E}u_n^p \le \int _{\{ u_n > h\}}u_n^p+h^p \mathrm{meas}(E). \end{aligned}$$

Then, thanks to (3.15), we take the limit as \(\mathrm{meas}(E)\) tends to zero, h tends to infinity, and since \( u_n^{p}\) converges to \(u^p\) almost everywhere, we easily conclude by Vitali’s theorem the proof of Proposition 3.4.

To conclude the proof of Theorem 1.1, we need to prove the strong convergence of \(T_k(u_n)\) to \(T_k(u)\) in \(W_\mathrm{loc}^{1, r}(\Omega )\). First, we choose \((T_k(u_n)+\varepsilon )^\theta -\varepsilon ^\theta \) as test function in (2.1) with \(\varepsilon < \frac{1}{n}\), using (1.2), dropping the positive term and by means of Hölder inequality and (3.11), we get:

$$\begin{aligned}&\alpha \int _{\Omega } \frac{|\nabla T_k(u_n)|^2}{(T_k(u_n)+\varepsilon )^{1-\theta }} \le B\int _{\Omega } |\nabla u_n|^q+ (k+\varepsilon )^\theta \int _{\Omega } f_n\\&\quad \le C\Vert u_n \Vert ^q_{W_0^{1, r}(\Omega )}+(k+\varepsilon )^\theta \Vert f \Vert _{L^1(\Omega )}. \end{aligned}$$

Then, from the previous inequality, we deduce that:

$$\begin{aligned}&\int _{\Omega } |\nabla T_k(u_n)|^2=\int _{\Omega } \frac{|\nabla T_k(u_n)|^2}{(T_k(u_n)+\varepsilon )^{1-\theta }} (T_k(u_n)+\varepsilon )^{1-\theta }\\&\quad \le (k+\varepsilon )^{1-\theta } (C\Vert u_n \Vert ^q_{W_0^{1, r}(\Omega )}+(k+\varepsilon )^\theta \Vert f \Vert ^q_{L^1(\Omega )}). \end{aligned}$$

Letting \(\varepsilon \) goes to zero:

$$\begin{aligned} \int _{\Omega } |\nabla T_k(u_n)|^2 \le C_ k, \end{aligned}$$
(3.16)

which implies that:

$$\begin{aligned} T_k(u_n) \ \text {converges to} \ T_k(u) \ \text {weakly} \ \text {in} \ H_0^{1}(\Omega ) \ \text {and a.e. in} \ \Omega . \end{aligned}$$
(3.17)

Then, thanks to (3.12), the right-hand side of (2.1) is bounded in \(L_\mathrm{loc}^1(\Omega )\), and by the result of [8] (see also [6, 9]), we deduce that, up to subsequences (not relabeled), \( \nabla u_n\) converges to \(\nabla u\) a.e. in \(\Omega \) which, in turn, implies that \(\frac{|\nabla u|^q}{u^\theta }\) belongs to \(L_\mathrm{loc}^1(\Omega )\) and that \(\frac{|\nabla u|^q}{u^\theta }\) belongs to \(L^1(\Omega )\) if \(0<\theta \le \frac{q}{q+1}\).

Next, let \(\varphi \in C^1_c(\Omega )\), \(\varphi \ge 0\), \(\varphi \equiv 1\) on \(\omega \subset \subset \Omega \) and use \(T_h(u_n-T_k(u))\varphi \) as test function in (2.1), we thus have thanks to (1.2), (2.5), and (3.11):

$$\begin{aligned}&\alpha \int _{\Omega } |\nabla T_h(u_n-T_k(u))|^2 \ \varphi \le C h \Vert \nabla \varphi \Vert _{L^{\infty }(\Omega )}\Vert u_n \Vert _{W_0^{1, r}(\Omega )}\\&\quad + h \Vert \varphi \Vert _{L^{\infty }(\Omega )} \ \frac{C}{c_\omega ^\gamma }\Vert u_n \Vert ^q_{W_0^{1, r}(\Omega )} \\&\quad +h \Vert \varphi \Vert _{L^{\infty }(\Omega )} \int _{\Omega } f-\int _{\Omega } M(x) \nabla T_k(u) \nabla T_h(u_n-T_k(u)) \ \varphi . \end{aligned}$$

Since \(T_h(u_n-T_k(u))\) converges to \(T_h(u-T_k(u))\) weakly in \(H^1_0(\Omega )\), by (1.2) and using the fact that \(\nabla T_k(u) \nabla T_h(u-T_k(u))\equiv 0\), we obtain:

$$\begin{aligned} \int _{\Omega } M(x) \nabla T_k(u) \nabla T_h(u_n-T_k(u)) \ \varphi =\omega (n). \end{aligned}$$

Hence:

$$\begin{aligned} \alpha \limsup _{n \rightarrow \infty } \int _{\Omega }|\nabla T_h(u_n-T_k(u))|^2 \ \varphi \le Ch. \end{aligned}$$
(3.18)

Let now r be such that \(r<2\), where r is as in the statement of Lemma 3.1, we can write:

$$\begin{aligned}&\int _{\omega }|\nabla T_k(u_n)-\nabla T_k(u)|^r\le \int _{\Omega }|\nabla T_k(u_n)-\nabla T_k(u)|^r \varphi \nonumber \\&\quad =\int _{\{ |u_n-u| \le h, u_n\le k\}}|\nabla u_n-\nabla T_k(u)|^r\varphi \nonumber \\&\qquad +\int _{\{ |u_n-u| \le h, u _n>k\}}|\nabla T_k(u)|^r\varphi \nonumber \\&\qquad +\int _{\{ |u_n-u| > h\}}|\nabla T_k(u_n)-\nabla T_k(u)|^r\varphi . \end{aligned}$$
(3.19)

Therefore, using Hölder inequality, we obtain:

$$\begin{aligned}&\int _{\Omega }|\nabla T_k(u_n)-\nabla T_k(u)|^r \varphi \le \int _{\Omega }|\nabla T_h(u_n-T_k(u))|^r\varphi \\&\quad +\Vert \varphi \Vert _{L^{\infty }(\Omega )} \Bigg (\int _{\{u _n>k\}}|\nabla T_k(u)|^r\\&\quad +(\mathrm{meas}{\{ |u_n-u| > h\}})^{1-\frac{r}{2}} \Bigg (\int _{\Omega }|\nabla T_k(u_n)-\nabla T_k(u)|^2\Bigg )^{\frac{r}{2}}\Bigg ) . \end{aligned}$$

Thus, combining (3.18) and (3.19) and thanks to (3.16) and Lebesgue’s convergence theorem, we obtain for every \(h>0\) and every \(k>0\):

$$\begin{aligned} \limsup _{n \rightarrow \infty } \int _{\Omega }|\nabla T_k(u_n)-\nabla T_k(u)|^r\ \varphi \le \Bigg (\frac{Ch}{\alpha }\Bigg )^{\frac{r}{2}} \Vert \varphi \Vert _{L^{\infty }(\Omega )} \mathrm{meas}(\Omega )^{1-\frac{r}{2}}, \end{aligned}$$

Letting h tends to zero, we finally have:

$$\begin{aligned} \limsup _{n \rightarrow \infty } \int _{\Omega }|\nabla T_k(u_n)-\nabla T_k(u)|^r\ \varphi =0. \end{aligned}$$

On the other hand, using \(1-(1+G_k(u_n))^{1-\lambda }\) as test function in (2.1) with \(\lambda >1\) and following the same proof of that (3.3) and using Hölder inequality, we obtain:

$$\begin{aligned} \int _{\{ u_n\ge k\}}|\nabla u_n|^r \le w(k). \end{aligned}$$
(3.20)

Hence:

$$\begin{aligned} u_n \ \text {converges to} \ u \ \text {strongly} \ \text {in} \ W_\mathrm{loc}^{1, r}(\Omega ). \end{aligned}$$
(3.21)

Now, let \(E\subset \subset \omega \) be a measurable set, and using (2.5) and Hölder inequality, we obtain:

$$\begin{aligned} \int _{E}\frac{|\nabla u_n|^q}{u_n^{\theta }}\le & {} \int _{E}\frac{|\nabla u_n|^q}{u_n^{\theta }} \varphi \le \frac{\Vert \varphi \Vert _{L^{\infty }(\Omega )} }{k^\theta } \int _{\{ u_n> k\}}|\nabla u_n|^q+\frac{\Vert \varphi \Vert _{L^{\infty }(\Omega )} }{c_\omega ^\theta } \int _{E}|\nabla u_n|^q\\\le & {} \frac{\Vert \varphi \Vert _{L^{\infty }(\Omega )} }{k^\theta } (\mathrm{meas}{\{ u_n> k\}})^{1-\frac{q}{r}} \Bigg (\int _{\{ u_n > k\}}|\nabla u_n|^r\Bigg )^{\frac{q}{r}}\\&+\frac{\Vert \varphi \Vert _{L^{\infty }(\Omega )} }{c_\omega ^\theta } \mathrm{meas}(E)^{1-\frac{q}{r}} \Bigg (\int _{E}|\nabla u_n|^r\Bigg )^{\frac{q}{r}}. \end{aligned}$$

Taking the limit as \(\mathrm{meas}(E)\) tends to zero, k tends to infinity, using (3.20) and (3.21), and since \(\frac{|\nabla u_n|^q}{u_n^{\theta }}\) converges to \(\frac{|\nabla u|^q}{u^{\theta }}\) almost everywhere, we easily verify thanks to Vitali’s theorem that:

$$\begin{aligned} \frac{|\nabla u_n|^q}{u_n^{\theta }} \text { converge strongly to} \ \ \frac{|\nabla u|^q}{u^{\theta }}\ \text {in} \ L_\mathrm{loc}^1(\Omega ). \end{aligned}$$
(3.22)

Now, we take \(\varphi \in C^1_{c}(\Omega )\) as test function in (2.1), and we have:

$$\begin{aligned} \int _{\Omega }M(x)\nabla u_n \nabla \varphi + \gamma \int _{\Omega } u_n^p \varphi = B\int _{\Omega } \frac{|\nabla u_n|^q}{(\frac{1}{n}|\nabla u_n|^q+1)(u_n+\frac{1}{n}))^\theta }\varphi +\int _{\Omega }f _n\varphi ; \end{aligned}$$

using (3.22), (3.21), and Proposition 3.4, we can pass to the limit with respect to n to conclude the proof of Theorem 1.1. \(\square \)